Quantcast
  • Register
PhysicsOverflow is a next-generation academic platform for physicists and astronomers, including a community peer review system and a postgraduate-level discussion forum analogous to MathOverflow.

Welcome to PhysicsOverflow! PhysicsOverflow is an open platform for community peer review and graduate-level Physics discussion.

Please help promote PhysicsOverflow ads elsewhere if you like it.

News

PO is now at the Physics Department of Bielefeld University!

New printer friendly PO pages!

Migration to Bielefeld University was successful!

Please vote for this year's PhysicsOverflow ads!

Please do help out in categorising submissions. Submit a paper to PhysicsOverflow!

... see more

Tools for paper authors

Submit paper
Claim Paper Authorship

Tools for SE users

Search User
Reclaim SE Account
Request Account Merger
Nativise imported posts
Claim post (deleted users)
Import SE post

Users whose questions have been imported from Physics Stack Exchange, Theoretical Physics Stack Exchange, or any other Stack Exchange site are kindly requested to reclaim their account and not to register as a new user.

Public \(\beta\) tools

Report a bug with a feature
Request a new functionality
404 page design
Send feedback

Attributions

(propose a free ad)

Site Statistics

205 submissions , 163 unreviewed
5,054 questions , 2,207 unanswered
5,345 answers , 22,721 comments
1,470 users with positive rep
818 active unimported users
More ...

  Hermitian conjugate of quasi-primary operator in 2D CFT

+ 2 like - 0 dislike
510 views

In all of the sources I could find, the hermitian conjugate of a quasi-primary operator $\mathcal{O}$ with weights $(h,\bar{h})$ in (2D Euclidean) CFT is given as

$$ \mathcal{O}(z,\bar{z})^\dagger = \bar{z}^{-2h} z^{-2\bar{h}} \mathcal{O}\left( \frac{1}{\bar{z}},\frac{1}{z} \right) $$

The justification is then, that we require operators to be self-adjoint in Lorentzian signature. The Euclidean time $x^0 = it$ is then inverted by complex conjugation, such that the coordinate $z = e^{x^0 + ix^1}$ is effectively inverted: $z \mapsto 1/\bar{z}$. But how does this justify the prefactors?

Since inversion is not an orientation preserving transformation (meaning $\partial_z (1/\bar{z}) = 0$), the usual formula for conformal transformation of a quasi primary field

$$ \mathcal{O}(z,\bar{z}) \mapsto \left( \frac{\partial f(z)}{\partial z} \right)^h \left( \frac{\partial \bar{f}(\bar{z})}{\partial \bar{z}} \right)^{\bar{h}} \mathcal{O}(f(z),\bar{f}(\bar{z})) $$

does not apply directly. However I would guess that one could compose such a transformation with a parity transformation (which I will take as complex conjugation (?)) to get the same effect. That is, first transform $z \mapsto 1/z$ and then perform an overall complex conjugation.

The first part would look like

$$ \mathcal{O}(z,\bar{z}) \mapsto \left( -\frac{1}{z^2} \right)^h \left( -\frac{1}{\bar{z}^2} \right)^{\bar{h}} \mathcal{O}\left( \frac{1}{z}, \frac{1}{\bar{z}} \right) $$

and performing an overall complex conjugation would then result in 

$$ \mathcal{O}(z,\bar{z}) \mapsto \left( -\frac{1}{\bar{z}^2} \right)^h \left( -\frac{1}{z^2} \right)^{\bar{h}} \mathcal{O}\left( \frac{1}{\bar{z}}, \frac{1}{z} \right). $$

Evidently this is still not the result that we want. What happens to the sign in the bracket? Why does no one even bother to mention it (from what I've been able to find)? Was my assumption, that we just need to conjugate the whole expression in the last step false?

asked May 27, 2022 in Theoretical Physics by justsomethingidc (15 points) [ no revision ]

Your answer

Please use answers only to (at least partly) answer questions. To comment, discuss, or ask for clarification, leave a comment instead.
To mask links under text, please type your text, highlight it, and click the "link" button. You can then enter your link URL.
Please consult the FAQ for as to how to format your post.
This is the answer box; if you want to write a comment instead, please use the 'add comment' button.
Live preview (may slow down editor)   Preview
Your name to display (optional):
Privacy: Your email address will only be used for sending these notifications.
Anti-spam verification:
If you are a human please identify the position of the character covered by the symbol $\varnothing$ in the following word:
p$\hbar$ysicsOve$\varnothing$flow
Then drag the red bullet below over the corresponding character of our banner. When you drop it there, the bullet changes to green (on slow internet connections after a few seconds).
Please complete the anti-spam verification




user contributions licensed under cc by-sa 3.0 with attribution required

Your rights
...